K
Khách

Hãy nhập câu hỏi của bạn vào đây, nếu là tài khoản VIP, bạn sẽ được ưu tiên trả lời.

6 tháng 5 2016

\(y'=\frac{1-\ln x-\left(1-\ln x-1\right)}{x^2\left(1-\ln x\right)^2}=\frac{1}{x^2\left(1-\ln x\right)^2}\)

5 tháng 5 2016

Ta có : \(y'=\frac{-1-\frac{1}{x}}{\left(1+x+\ln x\right)^2}=-\frac{x+1}{x\left(1+x+\ln x\right)^2}\) 

        \(\Rightarrow xy'=-\frac{x+1}{\left(1+x+\ln x\right)^2}\)    (1)

Lại có \(y\left(y\ln x-1\right)=\frac{-1-x}{\left(1+x+\ln x\right)^2}\)   (2)

Từ (1) và (2) suy ra \(xy'=y\left(y\ln x-1\right)\)

HQ
Hà Quang Minh
Giáo viên
22 tháng 9 2023

a) Với x > 0 bất kì và \(h = x - {x_0}\) ta có

\(\begin{array}{l}f'\left( {{x_0}} \right) = \mathop {\lim }\limits_{h \to 0} \frac{{f\left( {{x_0} + h} \right) - f\left( {{x_0}} \right)}}{h} = \mathop {\lim }\limits_{h \to 0} \frac{{\ln \left( {{x_0} + h} \right) - \ln {x_0}}}{h}\\ = \mathop {\lim }\limits_{h \to 0} \frac{{\ln \left( {1 + \frac{h}{{{x_0}}}} \right)}}{{\frac{h}{{{x_0}}}.{x_0}}} = \mathop {\lim }\limits_{h \to 0} \frac{1}{{{x_0}}}.\mathop {\lim }\limits_{h \to 0} \frac{{\ln \left( {1 + \frac{h}{{{x_0}}}} \right)}}{{\frac{h}{{{x_0}}}}} = \frac{1}{{{x_0}}}\end{array}\)

Vậy hàm số \(y = \ln x\) có đạo hàm là hàm số \(y' = \frac{1}{x}\)

b) Ta có \({\log _a}x = \frac{{\ln x}}{{\ln a}}\) nên \(\left( {{{\log }_a}x} \right)' = \left( {\frac{{\ln x}}{{\ln a}}} \right)' = \frac{1}{{x\ln a}}\)

12 tháng 5 2016

Ta có : \(y=\frac{1}{1+x+\ln x}\Rightarrow y'=\frac{-\left(1+\frac{1}{x}\right)}{\left(1+x+\ln x\right)^2}=\frac{-\left(1+x\right)}{x\left(1+x+\ln x\right)^2}\)

\(\Rightarrow\begin{cases}xy'=\frac{-\left(1+x\right)}{\left(1+x+\ln x\right)^2}\\y\left(y\ln x-1\right)=\frac{1}{1+x+\ln x}\left(\frac{\ln}{1+x+\ln x}-1\right)=\frac{-\left(1+x\right)}{\left(1+x+\ln x\right)^2}\end{cases}\)

\(\Rightarrow xy'=y\left(y\ln x-1\right)\Rightarrow\) Điều phải chứng minh

a: \(y'=\left(x^2+2x\right)'\left(x^3-3x\right)+\left(x^2+2x\right)\left(x^3-3x\right)'\)

\(=\left(2x+2\right)\left(x^3-3x\right)+\left(x^2+2x\right)\left(3x^2-3\right)\)

\(=2x^4-6x^2+2x^3-6x+3x^4-3x^2+6x^3-6x\)

\(=5x^4+8x^3-9x^2-12x\)

b: y=1/-2x+5 

=>\(y'=\dfrac{2}{\left(2x+5\right)^2}\)

c: \(y'=\dfrac{\left(4x+5\right)'}{2\sqrt{4x+5}}=\dfrac{4}{2\sqrt{4x+5}}=\dfrac{2}{\sqrt{4x+5}}\)

d: \(y'=\left(sinx\right)'\cdot cosx+\left(sinx\right)\cdot\left(cosx\right)'\)

\(=cos^2x-sin^2x=cos2x\)

e: \(y=x\cdot e^x\)

=>\(y'=e^x+x\cdot e^x\)

f: \(y=ln^2x\)

=>\(y'=\dfrac{\left(-1\right)}{x^2}=-\dfrac{1}{x^2}\)

Chọn D. Bởi vì hàm số ln x luôn luôn dương nên chắc chắn sẽ đồng biến trên TXĐ của nó

HQ
Hà Quang Minh
Giáo viên
24 tháng 8 2023

\(a,A=ln\left(\dfrac{x}{x-1}\right)+ln\left(\dfrac{x+1}{x}\right)-ln\left(x^2-1\right)\\ =ln\left(\dfrac{x}{x-1}\cdot\dfrac{x+1}{x}\right)-ln\left(x^2-1\right)\\ =ln\left(\dfrac{x+1}{x-1}\right)-ln\left(x^2-1\right)\\ =ln\left(\dfrac{x+1}{x-1}\cdot\dfrac{1}{x^2-1}\right)\\ =ln\left[\dfrac{1}{\left(x-1\right)^2}\right]\\ =2ln\left(\dfrac{1}{x-1}\right)\)

\(b,21log_3\sqrt[3]{x}+log_3\left(9x^2\right)-log_3\left(9\right)\\ =7log_3\left(x\right)+log_3x^2+log_39-log_39\\ =7log_3x+2log_3x\\ =9log_3x\)

HQ
Hà Quang Minh
Giáo viên
22 tháng 9 2023

a)

\(\begin{array}{c}A = {\log _{\frac{1}{3}}}5 + 2{\log _9}25 - {\log _{\sqrt 3 }}\frac{1}{5} = {\log _{{3^{ - 1}}}}5 + 2{\log _{{3^2}}}{5^2} - {\log _{{3^{\frac{1}{2}}}}}{5^{ - 1}}\\ =  - {\log _3}5 + 2{\log _3}5 + 2{\log _3}5 = 3{\log _3}5\end{array}\)                                     

b) \(B = {\log _a}{M^2} + {\log _{{a^2}}}{M^4} = 2{\log _a}M + \frac{1}{2}.4{\log _a}M = 4{\log _a}M\)

HQ
Hà Quang Minh
Giáo viên
22 tháng 9 2023

a) Với bất kì \({x_0} \in \mathbb{R}\), ta có:

\(f'\left( {{x_0}} \right) = \mathop {\lim }\limits_{x \to {x_0}} \frac{{f\left( x \right) - f\left( {{x_0}} \right)}}{{x - {x_0}}} = \mathop {\lim }\limits_{x \to {x_0}} \frac{{{e^x} - {e^{{x_0}}}}}{{x - {x_0}}}\)

Đặt \(x = {x_0} + \Delta x\). Ta có:

\(\begin{array}{l}f'\left( {{x_0}} \right) = \mathop {\lim }\limits_{\Delta x \to 0} \frac{{{e^{{x_0} + \Delta x}} - {e^{{x_0}}}}}{{\Delta x}} = \mathop {\lim }\limits_{\Delta x \to 0} \frac{{{e^{{x_0}}}.{e^{\Delta x}} - {e^{{x_0}}}}}{{\Delta x}} = \mathop {\lim }\limits_{\Delta x \to 0} \frac{{{e^{{x_0}}}.\left( {{e^{\Delta x}} - 1} \right)}}{{\Delta x}}\\ &  = \mathop {\lim }\limits_{\Delta x \to 0} {e^{{x_0}}}.\mathop {\lim }\limits_{\Delta x \to 0} \frac{{{e^{\Delta x}} - 1}}{{\Delta x}} = {e^{{x_0}}}.1 = {e^{{x_0}}}\end{array}\)

Vậy \({\left( {{e^x}} \right)^\prime } = {e^x}\) trên \(\mathbb{R}\).

b) Với bất kì \({x_0} > 0\), ta có:

\(f'\left( {{x_0}} \right) = \mathop {\lim }\limits_{x \to {x_0}} \frac{{f\left( x \right) - f\left( {{x_0}} \right)}}{{x - {x_0}}} = \mathop {\lim }\limits_{x \to {x_0}} \frac{{\ln {\rm{x}} - \ln {{\rm{x}}_0}}}{{x - {x_0}}}\)

Đặt \(x = {x_0} + \Delta x\). Ta có:

\(\begin{array}{l}f'\left( {{x_0}} \right) = \mathop {\lim }\limits_{\Delta x \to 0} \frac{{\ln \left( {{x_0} + \Delta x} \right) - \ln {{\rm{x}}_0}}}{{\Delta x}} = \mathop {\lim }\limits_{\Delta x \to 0} \frac{{\ln \left( {\frac{{{x_0} + \Delta x}}{{{{\rm{x}}_0}}}} \right)}}{{\Delta x}} = \mathop {\lim }\limits_{\Delta x \to 0} \frac{{\ln \left( {1 + \frac{{\Delta x}}{{{{\rm{x}}_0}}}} \right)}}{{\Delta x}}\\ = \mathop {\lim }\limits_{\Delta x \to 0} \frac{1}{{{x_0}}}.\frac{{\ln \left( {1 + \frac{{\Delta x}}{{{{\rm{x}}_0}}}} \right)}}{{\frac{{\Delta x}}{{{x_0}}}}} = \mathop {\lim }\limits_{\Delta x \to 0} \frac{1}{{{x_0}}}.\mathop {\lim }\limits_{\Delta x \to 0} \frac{{\ln \left( {1 + \frac{{\Delta x}}{{{{\rm{x}}_0}}}} \right)}}{{\frac{{\Delta x}}{{{x_0}}}}}\end{array}\)

Đặt \(\frac{{\Delta x}}{{{x_0}}} = t\). Lại có: \(\mathop {\lim }\limits_{\Delta x \to 0} \frac{1}{{{x_0}}} = \frac{1}{{{x_0}}};\mathop {\lim }\limits_{\Delta x \to 0} \frac{{\ln \left( {1 + \frac{{\Delta x}}{{{{\rm{x}}_0}}}} \right)}}{{\frac{{\Delta x}}{{{x_0}}}}} = \mathop {\lim }\limits_{t \to 0} \frac{{\ln \left( {1 + t} \right)}}{t} = 1\)

Vậy \(f'\left( {{x_0}} \right) = \frac{1}{{{x_0}}}.1 = \frac{1}{{{x_0}}}\)

Vậy \({\left( {\ln x} \right)^\prime } = \frac{1}{x}\) trên khoảng \(\left( {0; + \infty } \right)\).

12 tháng 5 2016

Ta có \(y'=\frac{\frac{1}{x}x\left(1-\ln x\right)-\left[1-\ln x+x\left(-\frac{1}{x}\right)\right]\left(1+\ln x\right)}{x^2\left(1-\ln x\right)^2}=\frac{1-\ln x+\ln x\left(1+\ln x\right)}{x^2\left(1-\ln x\right)^2}=\frac{1+\ln^2x}{x^2\left(1-\ln x\right)^2}\)

\(\Rightarrow\begin{cases}2x^2y'=2x^2\frac{1+\ln^2x}{x^2\left(1-\ln x\right)^2}=\frac{2\left(1+\ln^2x\right)}{\left(1-\ln x\right)^2}\\x^2y^2+1=x^2\frac{1+\ln^2x}{x^2\left(1-\ln x\right)^2}+1=\frac{\left(1+\ln^2x\right)}{\left(1-\ln x\right)^2}+1=\frac{2\left(1+\ln^2x\right)}{\left(1-\ln x\right)^2}\end{cases}\)

\(\Rightarrow2x^2y'=x^2y^2+1\Rightarrow\) Điều phải chứng minh